Tải bản đầy đủ (.pdf) (41 trang)

Tính bất khả quy của đa thức với hệ số nguyên

Bạn đang xem bản rút gọn của tài liệu. Xem và tải ngay bản đầy đủ của tài liệu tại đây (349.23 KB, 41 trang )

ĐẠI HỌC THÁI NGUYÊN
TRƯỜNG ĐẠI HỌC KHOA HỌC
---------------------------

PHẠM THỊ THU TRANG

TÍNH BẤT KHẢ QUY
CỦA ĐA THỨC VỚI HỆ SỐ NGUYÊN

LUẬN VĂN THẠC SĨ TOÁN HỌC

THÁI NGUYÊN - 2019


ĐẠI HỌC THÁI NGUYÊN
TRƯỜNG ĐẠI HỌC KHOA HỌC
---------------------------

PHẠM THỊ THU TRANG

TÍNH BẤT KHẢ QUY
CỦA ĐA THỨC VỚI HỆ SỐ NGUYÊN
Chuyên ngành: Phương pháp Toán sơ cấp
Mã số: 8 46 01 13

LUẬN VĂN THẠC SĨ TOÁN HỌC
NGƯỜI HƯỚNG DẪN KHOA HỌC
GS.TS. Lê Thị Thanh Nhàn

THÁI NGUYÊN - 2019



Mục lục
Lời cảm ơn

2

Mở đầu

3

1 Tiêu chuẩn Eisenstein và tiêu chuẩn rút gọn theo module
một số nguyên tố
5
1.1 Tiêu chuẩn Eisenstein và một số mở rộng . . . . . . . . . . 6
1.2 Tiêu chuẩn rút gọn theo module một số nguyên tố và bài
toán ngược . . . . . . . . . . . . . . . . . . . . . . . . . . . 11
2 Giá
2.1
2.2
2.3
2.4

trị khả nghịch, giá trị nguyên tố và tính bất khả
Giá trị khả nghịch và tính bất khả quy . . . . . . . .
Giá trị nguyên tố và tính bất khả quy . . . . . . . . .
Một tiêu chuẩn mới về tính bất khả quy . . . . . . . .
Giá trị nguyên tố tại đối số đủ lớn và tính bất khả quy

quy
. . .

. . .
. . .
. . .

16
16
21
27
32

Kết luận

37

Tài liệu tham khảo

38

1


Lời cảm ơn
Trước tiên tôi xin gửi lời cảm ơn chân thành và sâu sắc nhất tới GS.TS
Lê Thị Thanh Nhàn. Mặc dù rất bận rộn trong công việc, song ngay từ
những ngày đầu tiên Cô đã luôn tận tình chỉ bảo, hướng dẫn và đưa ra
những lời khuyên có ích giúp tôi hoàn thiện luận văn này.
Tôi cũng xin gửi lời cảm ơn tới các thầy, cô cán bộ khoa Toán - Tin,
trường Đại học Khoa học - Đại học Thái Nguyên, Ban giám hiệu và các
đồng nghiệp trường Trung học phổ thông Hoành Bồ - Tỉnh Quảng Ninh
cùng các bạn tập thể lớp Cao học Toán K11D, đã không chỉ trang bị cho

tôi những kiến thức bổ ích mà còn luôn luôn giúp đỡ tôi, tạo điều kiện cho
tôi trong thời gian theo học tại trường.
Cuối cùng, tôi xin chân thành bày tỏ lòng biết ơn đến gia đình, bạn bè,
những người đã không ngừng ủng hộ, động viên, hỗ trợ và tạo mọi điều
kiện giúp tôi vượt qua những khó khăn để hoàn thiện luận văn.

2


Mở đầu
Tính bất khả quy của đa thức với hệ số nguyên trên trường các số phức
C và trên trường các số thực R đã được giải quyết từ thế kỷ 19 thông qua
Định lý cơ bản của Đại số. Tuy nhiên, tính bất khả quy của đa thức với
hệ số nguyên trên trường các số hữu tỷ Q đến nay vẫn đang thách thức
các nhà Toán học trên thế giới.
Trong luận văn này, tác giả trình bày lại một số tiêu chuẩn bất khả quy
của đa thức trên trường số hữu tỷ Q với hệ số nguyên trong các bài báo
gần đây [8] và [11].
Luận văn gồm 2 chương. Trong chương 1, chúng tôi trình bày hai tiêu
chuẩn bất khả quy nổi tiếng. Phần 1.1 trình bày Tiêu chuẩn Eisenstein và
các mở rộng. Phần 1.2 trình bày tiêu chuẩn rút gọn theo module một số
nguyên tố và phát biểu đảo của tiêu chuẩn này. Nội dung chương 1 được
viết theo bài báo [11] cuả R. Thangadurai năm 2007.
Chương 2 trình bày các tiêu chuẩn bất khả quy trên trường các số hữu
tỷ Q liên quan đến các giá trị khả nghịch và giá trị nguyên tố của đa thức
với hệ số nguyên. Phần 2.1 trình bày các tiêu chuẩn về sự liên quan giữa
giá trị khả nghịch với tính bất khả quy của đa thức. Phần 2.2 trình bày về
mối quan hệ giữa giá trị nguyên tố và tính bất khả quy. Các kết quả ở hai
phần này cũng được viết dựa theo bài báo [11] của R. Thangadurai năm
2007. Phần 2.3 trình bày một tiêu chuẩn bất khả quy mới trên trường Q

các số hữu tỷ liên quan đến đa thức có các hệ số nguyên tăng dần theo chỉ
số và có hệ số cao nhất nguyên tố hoặc nhận ít nhất một giá trị nguyên
tố. Kết quả của phần này được viết dựa theo bài báo [8] của A. Jakhar và
N. Sangwan năm 2018. Phần 2.4 trình bày về giá trị nguyên tố tại đối số
đủ lớn và tính bất khả quy của đa thức với hệ số nguyên. Nội dung của
phần này được viết trên cơ sở nội dung bài báo [11] của R. Thangadurai
năm 2007.
3


Trong luận văn này, các tiêu chuẩn trong các phần 2.1 về giá trị khả
nghịch và tính bất khả quy; phần 2.2 về giá trị nguyên tố và tính bất khả
quy; phần 2.3 về tiêu chuẩn mới cho tính bất khả quy là những kết quả
chưa được trình bày trong bất cứ luận văn thạc sĩ nào trước đây.
Hơn thế, trong các phần 1.1, 1.2, 2.4, mặc dù có một số kết quả đã
quen biết và được trình bày trong một vài luận văn trước đây (xem [1],
[2]), nhưng cách chứng minh và ví du hầu như là mới, do chính tác giả
luận văn tự tính toán. Đặc biệt nếu trong luận văn [2], Nguyễn Văn Lập
chứng minh đa thức x4 − 2x2 + 9 là bất khả quy trên Q nhưng không bất
khả quy trên Zp với mọi số nguyên tố p bằng cách sử dụng kiến thức về
nhóm, thì trong luận văn này chứng minh đa thức x4 + 1 bất khả quy trên
Q nhưng khả quy trên Zp với mọi số nguyên tố p bằng cách sử dụng kiến
thức về trường hữu hạn.
Thái Nguyên, ngày 25 tháng 5 năm 2019
Tác giả luận văn

Phạm Thị Thu Trang

4



Chương 1

Tiêu chuẩn Eisenstein và tiêu chuẩn
rút gọn theo module một số nguyên
tố
Một đa thức với hệ số trên một trường được gọi là bất khả quy nếu nó
có bậc dương và không phân tích được thành tích của hai đa thức có bậc
thấp hơn. Một đa thức bậc dương với hệ số trên một trường là khả quy
nếu nó là tích của hai đa thức với bậc thấp hơn.
Chú ý rằng tính bất khả quy của đa thức phụ thuộc vào trường cơ sở.
Chẳng hạn, đa thức x2 − 2 là bất khả quy trên trường Q các số hữu tỷ,
nhưng không bất khả quy trên trường R các số thực. Đa thức x2 + 1 bất
khả quy trên trường R nhưng không bất khả quy trên trường C các số
phức.
Tính bất khả quy trên trường các số phức và trên trường các số thực đã
được làm rõ nhờ Định lý cơ bản của Đại số: Mọi đa thức bậc dương với hệ
số phức đều có ít nhất một nghiệm phức. Vì thế các đa thức bất khả quy
trên C là và chỉ là các đa thức bậc nhất. Các đa thức bất khả quy trên R
là và chỉ là các đa thức bậc nhất hoặc đa thức bậc hai có biệt thức âm.
Câu hỏi được đặt ra là khi nào đa thức f (x) đã cho là khả quy hay bất
khả quy trên Q? Cho đến nay, không có điều kiện cần và đủ nào có thể áp
dụng được cho tất cả các đa thức, mà ta chỉ có một số tiêu chuẩn để kiểm
tra tính bất khả quy của một số trường hợp cụ thể.
Rõ ràng mọi đa thức bậc nhất đều bất khả quy trên Q. Các đa thức bậc
hai và bậc ba là bất khả quy trên Q nếu và chỉ nếu nó không có nghiệm
5


hữu tỷ. Đối với đa thức bậc lớn hơn 3, nếu đa thức có nghiệm hữu tỷ thì

nó không bất khả quy. Tuy nhiên điều ngược lại không đúng. Chẳng hạn,
đa thức (x2 + 1)2 không có nghiệm hữu tỷ, nhưng không bất khả quy.
Trong chương này, chúng tôi trình bày hai tiêu chuẩn nổi tiếng về tính
bất khả quy trên trường các số hữu tỷ Q của đa thức với hệ số nguyên
dựa theo bài báo [11] của R. Thangadurai. Phần thứ nhất dành để trình
bày Tiêu chuẩn Eisensrein và một số mở rộng của nó. Mở rộng thứ nhất
được phát hiện bởi H. Chao trong bài báo A Generalization of Eisenstein’s
Criterion, Mathematics Magazine, Vol. 47 (1974), 158-159 và mở rộng thứ
hai được đưa ra bởi S. H. Weintraub trong bài báo A mild generazation of
Eisenstein criterion, Proceedings of the American Mathematical Society,
Vol. 141 (2013), 1159-1160. Phần tiếp theo trình bày một trong những tiêu
chuẩn bất khả quy phổ biến nhất, đó là tiêu chuẩn rút gọn theo module
một số nguyên tố. Phát biểu đảo của tiêu chuẩn này không còn đúng nữa,
chúng tôi đưa ra một chứng minh chi tiết để minh họa điều này.

1.1

Tiêu chuẩn Eisenstein và một số mở rộng

Trong mục này, chúng tôi trình bày lại tiêu chuẩn Eisenstein và một số
mở rộng liên quan về tính bất khả quy của các đa thức với hệ số nguyên
trên trường các số hữu tỉ Q. Đây là một trong những tiêu chuẩn quen
thuộc thường được sử dụng khi làm các bài toán về tính bất khả quy của
đa thức trên Q.
Cho
f (x) = an xn + an−1 xn−1 + · · · + a1 x + a0
là đa thức bậc n với ai ∈ Z, an = 0.
Tiêu chuẩn bất khả quy được biết đến nhiều nhất hiện nay là tiêu chuẩn
Eisenstein, được phát biểu như sau.
1.1.1 Định lý 1. Cho đa thức f (x) = an xn + an−1 xn−1 + · · · + a1 x + a0

là đa thức với hệ số nguyên có bậc n > 0. Nếu tồn tại một số nguyên tố
p sao cho p an , p | ai với mọi i = 0, 1, . . . , n − 1 và p2 a0 , thì đa thức
f (x) bất khả quy trên Q.
Chứng minh. Giả sử f (x) khả quy trên Q. Theo Bổ đề Gauss, tồn tại biểu
6


diễn f (x) = g(x)h(x), trong đó g(x) = bm xm + · · · + b1 x + b0 ∈ Z[x] và
h(x) = ck xk + · · · + c1 x + c0 ∈ Z[x] với deg g(x) = m, deg h(x) = k và
m, k < n. Do p là ước của a0 = b0 c0 nên p | b0 hoặc p | c0 . Mặt khác, p2
không là ước của a0 nên trong hai số b0 và c0 , chỉ có một và chỉ một số chia
hết cho p. Giả thiết p | c0 . Khi đó b0 không chia hết cho p. Vì an = bm ck
và p an nên bm và ck đều không chia hết cho p. Do đó tồn tại số r bé
nhất sao cho cr không là bội của p. Ta có

ar = b0 cr + (b1 cr−1 + b2 cr−2 + · · · + br c0 ).
Vì r ≤ k < n nên p | ar . Theo cách chọn r ta có

p | b1 cr−1 + b2 cr−2 + · · · + br c0 .
Suy ra p | b0 cr , điều này là vô lí vì cả hai số b0 và cr đều không là bội của
p. Vậy f (x) là bất khả quy trên Q.
Các đa thức thỏa mãn Định lý 1 được gọi là đa thức Eisenstein. Chẳng
hạn, đa thức x5 − 4x4 + 18x3 + 24x2 + 4x + 6 là đa thức Eisenstein vì nó
bất khả quy theo Tiêu chuẩn Eisenstein với p = 2.
Thông thường, Tiêu chuẩn Eisenstein không áp dụng được trực tiếp cho
đa thức f (x), mà chúng ta có thể áp dụng cho đa thức f (x + a) với a
là hằng số nào đó. Chú ý rằng đa thức f (x) là bất khả quy trên Q nếu
và chỉ nếu đa thức f (x + a) là bất khả quy trên Q với mọi số nguyên a.
Do vậy, chúng ta cố gắng tìm hằng số a với hy vọng khi biến đổi đa thức
f (x + a) ta được một đa thức mới thỏa mãn các điều kiện của Tiêu chuẩn

Eisenstein. Dưới đây là một ví dụ về tính bất khả quy của đa thức chia
đường tròn thứ p với p là một số nguyên tố.
1.1.2 Ví dụ 1. Cho p là số nguyên tố. Khi đó đa thức chia đường tròn
thứ p
f (x) = xp−1 + xp−2 + · · · + x + 1
là bất khả quy trên Q.
Chứng minh. Đa thức f (x) = xp−1 + xp−2 + · · · + x + 1 có các hệ số đều
bằng 1 nên không thể áp dụng trực tiếp Tiêu chuẩn Eisenstein để xét tính
bất khả quy của f (x).

7


xp − 1
Chú ý rằng f (x) =
. Suy ra, chọn a = 1 ta có
x−1
(x + 1)p − 1
= xp−1 + Cp1 xp−2 + . . . + Cpp−2 x + Cpp−1 ,
f (x + 1) =
x
p!
trong đó Cpk =
là số tổ hợp chập k của p phần tử. Do p nguyên
(p − k)!k!
tố nên Cpk là bội của p với mọi k = 1, 2, . . . , p − 2 và Cpp−1 = p không là
bội của p2 . Vì vậy f (x + 1) là bất khả quy theo Tiêu chuẩn Eisenstein (áp
dụng cho số nguyên tố p). Do đó f (x) bất khả quy trên Q.
Như vậy, thông qua tiêu chuẩn Eisenstein, từ bài toán ban đầu về xét
tính bất khả quy của đa thức bậc n với hệ số nguyên, ta đưa về bài toán

phân tích n hệ số của đa thức mới f (x + a), sau khi biến đổi đa thức
f (x + a) cần tìm ra ước chung nguyên tố phù hợp của các hệ số, trừ hệ
số cao nhất, của đa thức f (x + a). Hiển nhiên, chúng ta cố gắng biến đổi
đa thức để tạo ra đa thức mới với hệ số lớn hơn, nhưng nhiệm vụ sau đó
là tính toán và kiểm tra các ước nguyên tố chung của các hệ số thỏa mãn
điều kiện trong Tiêu chuẩn Eisenstein. Tuy nhiên, chúng ta chưa chắc chắn
về sự tồn tại của phép biến đổi để đa thức ban đầu chuyển thành đa thức
mới có thể áp dụng tiêu chuẩn Eisenstein, tức là chưa chắc đã tìm được
số nguyên a để đa thức f (x + a) áp dụng được Tiêu chuẩn Eisenstein ứng
với một số nguyên tố p nào đó. Ví dụ, người ta đã chỉ ra rằng đa thức
x4 − 10x2 + 1 là bất khả quy trên Q nhưng không tìm được số nguyên a
để đa thức
(x + a)4 − 10(x + a)2 + 1
bất khả quy theo Tiêu chuẩn Eisenstein với một số nguyên tố p nào đó.
Trong phần cuối của mục này, chúng ta nhắc lại một số mở rộng của
Tiêu chuẩn Eisenstein. Trước hết chúng ta nhắc lại tiêu chuẩn bất khả
quy của H. Chao trong bài báo A Generalization of Eisenstein’s Criterion,
Mathematics Magazine, Vol. 47 (1974), 158-159.
1.1.3 Định lý 2. Cho f (x) = an xn + . . . + a1 x + a0 là đa thức bậc n với
hệ số nguyên. Giả sử p là một số nguyên tố sao cho có hai chỉ số t = k
thỏa mãn: p không là ước của at , p là ước của ai với mọi i = t và p2 không
là ước của ak . Khi đó nếu f (x) là tích của hai đa thức với hệ số nguyên,
thì một trong hai đa thức đó có bậc lớn hơn hoặc bằng | t − k |.
8


Chứng minh. Xem [1].
Trước khi đưa ra một số ví dụ minh họa cho việc áp dụng tiêu chuẩn
trong Định lý 2, chúng ta chú ý điều kiện về nghiệm hữu tỷ của đa thức
với hệ số nguyên như sau: Nếu r/s là phân số tối giản và là nghiệm của

đa thức f (x) với hệ số nguyên, thì r phải là ước của hệ số tự do và s là
ước của hệ số cao nhất.
Định lý trên là một mở rộng không tầm thường của Tiêu chuẩn Eisenstein. Chú ý rằng nếu f (x) là đa thức với hệ số nguyên phân tích được
thành tích của hai đa thức với hệ số hữu tỷ g(x) và h(x), thì nó phân tích
được thành tích của hai đa thức với hệ số nguyên g1 (x) và h1 (x), trong
đó deg g(x) = deg g1 (x) và deg h(x) = deg h1 (x), xem Bổ đề Gauss ([3,
Định lý 2.3.2]). Vì thế, khi t = n và k = 0, thì định lý trên trở thành Tiêu
chuẩn Eisenstein. Khi t = 0 và k = n thì mọi đa thức thỏa mãn điều kiện
trong định lý trên vẫn là đa thức bất khả quy trên Q.
1.1.4 Ví dụ 2. Các đa thức sau là bất khả quy trên Q.
(i) f (x) = 18x100 − 50x2 + 40x + 1.
(ii) g(x) = 322x4 + 256x2 + 5x + 2.
Chứng minh.
(i) Áp dụng Định lý 2 với t = 0, k = 100 và p = 2, ta suy ra f (x) bất khả
quy trên Q.
(ii) Áp dụng Định lý 2 với t = 1, k = 4 và p = 2, ta suy ra rằng nếu h(x)
là một đa thức với hệ số nguyên và là ước của g(x), thì h(x) phải có bậc
lớn hơn hoặc bằng 3 hoặc h(x) có bậc nhỏ hơn hoặc bằng 1. Dễ thấy rằng
nếu g(x) có nhân tử bậc 1 thì nó phải có nghiệm hữu tỷ, và nghiệm đó chỉ
có thể là 1, −1, 2, −2. Rõ ràng tất cả các số trên đều không là nghiệm của
g(x), vì thế nó không có nhân tử bậc 1. Suy ra h(x) có bậc 4 hoặc có bậc
0. Vì thế g(x) bất khả quy trên Q.
S. H. Weintraub trong bài báo: A mild generazation of Eisenstein criterion, Proceedings of the American Mathematical Society, Vol. 141 (2013),
1159-1160 đã đưa ra môt mở rộng của Tiêu chuẩn Eisenstein, được phát
biểu như sau.
1.1.5 Định lý 3. Cho f (x) = an xn + . . . + a1 x + a0 là đa thức bậc n với hệ
số nguyên. Giả sử p là một số nguyên tố sao cho p không là ước của an , p
9



là ước của ai với mọi i = n và p2 không là ước của ak với 0 ≤ k ≤ n − 1.
Gọi k0 là số bé nhất trong các số k thỏa mãn điều kiện trên. Khi đó nếu
f (x) = g(x)h(x) là tích của hai đa thức với hệ số nguyên, thì

min{deg g(x), deg h(x)} ≤ k0 .
Chứng minh. Xem [1].
Định lý trên cũng là một mở rộng không tầm thường của Tiêu chuẩn
Eisenstein. Khi k0 = 0, thì ta nhận được Tiêu chuẩn Eisenstein. Khi k0 = 1
và f (x) không có nghiệm hữu tỷ, thì f (x) cũng bất khả quy theo định lý
trên. Khi k0 = 2 và f (x) không có nhân tử bậc hai, thì f (x) cũng bất khả
quy.
1.1.6 Ví dụ 3. Đa thức f (x) = x4 − 14x2 + 4 là bất khả quy trên Q.
Chứng minh. Áp dụng Định lý 3 với n = 4, k0 = 2 và p = 2, ta suy ra rằng
nếu f (x) = g(x)h(x) là tích của hai đa thức với hệ số nguyên, thì g(x)
hoặc h(x) có bậc nhỏ hơn hoặc bằng 2. Không mất tính tổng quát ta giả
sử h(x) có bậc nhỏ hơn hoặc bằng 2. Xét trường hợp h(x) có bậc 1. Khi đó
f (x) có nghiệm hữu tỷ, và nghiệm đó chỉ có thể là 1, −1, 2, −2, 4, −4. Rõ
ràng tất cả các số trên đều không là nghiệm của f (x), vì thế h(x) không
thể có bậc 1. Giả sử h(x) có bậc 2. Theo Bổ đề Gauss, ta có thể viết

x4 − 14x2 + 4 = (x2 + ax + b)(x2 + cx + d)
với a, b, c, d là các số nguyên. Đồng nhất hệ số cả hai vế ta được

a + c = 0, b + d + ac = −14, ad + bc = 0, bd = 4.
Suy ra (b, d) chỉ có thể là một trong các cặp sau

(1, 4), (−1, −4), (2, 2), (−2, −2), (4, 1), (−4, −1).
Kiểm tra tất cả các trường hợp trên ta đều thấy hoặc các đẳng thức không
thỏa mãn, hoặc a không là số hữu tỷ. Vì thế h(x) chỉ có thể có bậc 0. Suy
ra f (x) bất khả quy trên Q.


10


1.2

Tiêu chuẩn rút gọn theo module một số nguyên tố và bài
toán ngược

Một tiêu chuẩn xét tính bất khả quy trên Q cũng rất quen biết, được
gọi là tiêu chuẩn rút gọn theo module một số nguyên tố.
Trước khi phát biểu tiêu chuẩn rất quen thuộc này, chúng ta cần nhắc
lại một số ký hiệu.
Cho n > 1 là một số tự nhiên. Ký hiệu Zn là vành các số nguyên
modulo n. Khi đó Zn là một trường (tức là mọi phần tử khác 0 trong Zn
đều có nghịch đảo) khi và chỉ khi n là số nguyên tố. Chẳng hạn, Z5 là một
trường, Z4 không là trường. Ta quy ước viết đa thức f (x) ∈ Zp [x], với p
là số nguyên tố, là đa thức thu được bằng cách chuyển hệ số của f (x) vào
trường Zp . Chẳng hạn, nếu f (x) = 10x2 + 8, thì f (x) = 3x2 + 1 ∈ Z7 [x].
Tiêu chuẩn rút gọn theo module một số nguyên tố được phát biểu như
sau.
1.2.1 Định lý 4. Cho f (x) là đa thức với hệ số nguyên. Nếu tồn tại
số nguyên tố p sao cho f (x) bất khả quy trên trường Zp và deg f (x) =
deg f (x), thì f (x) bất khả quy trên Q.
Chứng minh. Vì đa thức f (x) bất khả quy trên Zp nên deg f (x) > 0. Suy
ra deg f (x) > 0. Giả sử f (x) khả quy trên Q. Theo Bổ đề Gauss, f (x) có
phân tích f (x) = g(x)h(x) trong đó g(x), h(x) ∈ Z[x] và g(x), h(x) có bậc
nhỏ hơn bậc của f (x). Chú ý rằng f (x) = g(x)h(x). Do đó deg f (x) =
deg g(x) + deg h(x). Rõ ràng ta có deg g(x) ≥ deg g(x) và deg h(x) ≥
deg h(x). Do đó f (x) phân tích được thành tích của hai đa thức g(x), h(x)

có bậc thấp hơn. Điều này mâu thuẫn với tính bất khả quy của f (x) trên
Zp .
Tiêu chuẩn rút gọn theo module một số nguyên tố là một tiêu chuẩn
rất hữu hiệu để kiểm tra tính bất khả quy của đa thức với hệ số nguyên.
Có những đa thức có thể áp dụng trực tiếp tiêu chuẩn này, chẳng hạn với
các đa thức bậc ba, người ta thường rút gọn theo modulo một số nguyên
tố p rồi kiểm tra đa thức trong Zp [x] có nghiệm trong Zp hay không. Ví
dụ, đa thức
f (x) = x3 + 591x2 + 3801 + 24240
11


là bất khả quy trên Q. Thật vậy, trong vành Z2 [x], đa thức f (x) = x3 +
x2 + 1 không có nghiệm trong Z2 , vì thế đa thức f (x) bất khả quy trên
Z2 . Do deg f (x) = 3 = deg f (x), nên f (x) bất khả quy trên Q theo Định
lý 1.2.1. Chú ý rằng việc kiểm tra nghiệm hữu tỷ của đa thức f (x) ở trên
là vấn đề không khả thi bằng các công cụ thông thường.
1.2.2 Ví dụ 4. Xét tính bất khả quy của đa thức f (x) = 5x2 + 20x + 19.
Chứng minh. Vì f (x) = 2x2 + 2x + 1 ∈ Z3 [x] không có nghiệm trong Z3 và
deg f (x) = 2 nên f (x) bất khả quy trên Z3 . Rõ ràng deg f (x) = deg f (x)
nên f (x) bất khả quy trên Q theo Định lý 1.2.1.
1.2.3 Ví dụ 5. Xét tính bất khả quy của đa thức sau

g(x) = 6x4 + 10x3 − 9x2 + 11x + 1.
Chứng minh. Vì g(x) = x4 + x2 + x + 1 ∈ Z5 [x] không có nghiệm trong Z5
nên nó không có nhân tử bậc một. Giả sử g(x) khả quy trên Z5 . Khi đó

g(x) = (x2 + ax + b)(x2 + cx + d)
với a, b, c, d ∈ Z5 . Đồng nhất hệ số ở hai vế của đẳng thức này ta được


a + c = 0, b + ac + d = 1, ad + bc = 1, bd = 1.
Vì bd = 1 và vai trò của b, d là như nhau nên không mất tính tổng quát
ta có thể giả thiết (b, d) = (1, 1) hoặc (b, d) = (2, 3) hoặc (b, d) = (4, 4).
Nếu (b, d) = (1, 1) thì các phương trình đầu và cuối cho ta a + c = 0 và
a + c = 1, vô lí. Nếu (b, d) = (2, 3) thì các phương trình đầu và cuối cho
ta a = 1, c = 4, và do đó phương trình thứ hai cho ta 4 = ac = 1, vô lí.
Nếu (b, d) = (4, 4) thì các phương trình đầu và cuối cho ta a + c = 0 và
4(a + c) = 1, vô lí. Vì vậy h(x) bất khả quy trên Z5 . Vì deg h(x) = 4 =
deg h(x) nên theo Định lý 1.2.1 đa thức h(x) bất khả quy trên Q.
Điều ngược lại của Định lý 4 là không đúng, nghĩa là, nếu f (x) bất khả
quy trên Q thì chưa chắc nó đã bất khả quy trên Zp với một số nguyên tố
p nào đó. D. Hilbert là người đầu tiên chỉ ra ví dụ về một đa thức với hệ
số nguyên bất khả quy trên Q nhưng không bất khả quy trên Zp với mọi
số nguyên tố p. Trong luận văn thạc sĩ của Nguyễn Văn Lập (xem [2]) đã
đưa ra chứng minh chi tiết rằng đa thức x4 − 2x2 + 9 là bất khả quy trên
12


Q nhưng không bất khả quy trên Zp với mọi số nguyên tố p. Chứng minh
trình bày trong [2] phải sử dụng những kiến thức khá sâu về lý thuyết
nhóm.
Trong luận văn này, chúng tôi làm rõ kết quả của D. Hilbert bằng cách
chỉ ra rằng đa thức x4 + 1 bất khả quy trên Q, nhưng không bất khả quy
trên Zp với mọi p nguyên tố. Chứng minh kết quả này dựa theo bài báo
[8] bằng cách sử dụng những kiến thức về mở rộng trường. Vì thế, trước
hết chúng ta cần trình bày một số kiến thức về mở rộng trường.
1.2.4 Định nghĩa 1. Cho K là một trường và F là một trường chứa K .
Khi đó ta nói F là môt mở rộng trường của K và ta viết là F/K . Xét F
như một không gian vec tơ trên trường K . Nếu chiều của K -không gian
véc tơ F là n thì ta nói mở rộng trường F/K có bậc n.

Chẳng hạn, cho K = Q và


F = Q[ 2] = {a + b 2 | a, b ∈ Q}.

Khi đó F là K -không gian véc tơ chiều là 2 với một cơ sở là {1, 2}. Vì
thế bậc của mở rộng F/K là 2.
1.2.5 Mệnh đề 1. Cho K là một trường và f (x) ∈ K[x] là một đa thức
bất khả quy trên K . Cho deg f (x) = n và α là một nghiệm trong một mở
rộng trường nào đó của K . Khi đó

K[α] = {g(α) | g(x) ∈ K[x]}
là một mở rộng trường của K , bậc của mở rộng là n và hệ {1, α, . . . , αn−1 }
là một cơ sở của K[α].
Chứng minh. . Xem [3, Mệnh đề 2.4.2]
Ví dụ, cho K = Q và f (x) = x5 − 2. Khi đó f (x) bất khả quy trên
Q theo Tiêu chuẩn Eisenstein với p = 2. Gọi α là môt nghiệm của f (x)
trong C (chú ý rằng f (x) luôn có nghiệm trong C theo Định lý cơ bản của
Đại số). Khi đó Q[α] là một mở rộng bậc 5 của Q và hệ {1, α, α2 , α3 , α4 }
là một cơ sở của Q[α].
1.2.6 Mệnh đề 2. Cho K là một trường và f (x) ∈ K[x]. Khi đó tồn tại
duy nhất một trường tối thiểu chứa K và chứa tất cả các nghiệm của f (x).
Chứng minh. Xem [3, Định lý 2.4.7]
13


Cho K là một trường và f (x) ∈ K[x] là đa thức có bậc n. Trường tối
thiểu chứa K và chứa đủ n nghiệm của f (x) (luôn tồn tại theo mệnh đề
trên) được gọi là trường phân rã của f (x) trên K . Ví dụ, cho K = R và
f (x) = x2 + 1. Khi đó f (x) bất khả quy trên R. Các nghiệm của f (x)

là i và −i. Do đó C là trường tối thiểu chứa R và chứa các nghiệm của
f (x), nói cách khác C là trường phân rã của f (x) trên R. Cho K = Q và

f (x) = x2 − 2. Khi đó Q[ 2] là trường tối thiểu chứa Q và các nghiệm
của f (x), do đó nó là trường phân rã của f (x) trên Q.
Kết quả dưới đây cho ta cấu trúc của trường hữu hạn.
1.2.7 Mệnh đề 3. Các phát biểu sau là đúng.
(i) Nếu K là trường hữu hạn có q phần tử thì q là lũy thừa của một số
nguyên tố.
(ii) Nếu q là lũy thừa của một số nguyên tố, thì tồn tại duy nhất một
trường có q phần tử.
(iii) Giả sử q = pk với k là một số tự nhiên và p là số nguyên tố. Khi đó
k
trường phân rã của đa thức xp − x ∈ Zp [x] chính là tâp tất cả các
k
nghiệm của đa thức xp − x trong một mở rộng nào đó của Zp .
Chứng minh. Xem [3, Mệnh đề 2.4.10].
Bây giờ chúng ta sử dụng các kết quả trên về mở rộng trường để chứng
minh khẳng định của D. Hilbert về sự tồn tại một đa thức với hệ số nguyên
bất khả quy trên Q, nhưng khả quy trên mọi trường Zp với p nguyên tố.
1.2.8 Định lý 5. Đa thức f (x) = x4 + 1 bất khả quy trên Q nhưng khả
quy trên Zp với mọi số nguyên tố p.
Chứng minh. Với p = 2, rõ ràng f (x) = x4 + 1 ≡ (x2 + 1)2 ∈ Z2 [x]. Suy
ra đa thức f (x) khả quy trên Z2 .
Cho p ≥ 3 là số nguyên tố bất kỳ. Khi đó, p là số lẻ. Viết p = 2k + 1,
ta có p2 − 1 = 4k(k + 1) là số chia hết cho 8. Chú ý rằng nếu n là ước của
m với n, m là hai số nguyên dương, thì xn − 1 là ước của xm − 1. Suy ra

(x8 − 1) | (xp


2

−1

− 1).

Nhân cả hai vế với x ta suy ra
2

x(x4 + 1)(x4 − 1) | (xp − x).
14


2

Vì thế f (x) = x4 + 1 là ước của đa thức xp − x. Ký hiệu Fp2 là trường
2
phân rã của đa thức xp − x trên trường Zp (luôn tồn tại theo Mệnh để 2).
2
Khi đó Fp2 chính là tập nghiệm của đa thức xp − x trong một mở rộng
nào đó của Zp (xem Mệnh đề 3). Vì Fp2 có p2 phần tử và Zp có p phần tử
nên Fp2 là Zp -không gian véc tơ chiều 2.
Ta chứng minh Định lý bằng phương pháp phản chứng.
Giả sử f (x) = x4 + 1 bất khả quy trên K := Zp với số nguyên tố p ≥ 3
nào đó. Ta cần tìm mâu thuẫn.
Gọi α là một nghiệm của f (x). Khi đó α cũng là nghiệm của đa thức
2
p2
x − x. Vì thế α là phần tử của trường phân rã Fp2 của đa thức xp − x
trên K . Đặt K1 = K[α]. Khi đó K1 là trường trung gian giữa K và Fp2 .

Vì f (x) bất khả quy trên K và deg f (x) = 4, nên theo Mệnh đề 1, K1 là
một không gian véc tơ có chiều bằng 4 trên K . Như vậy, không gian véc
tơ con K1 là K - không gian véc tơ chiều 4, trong khi đó không gian véc tơ
chứa K1 là Fp2 lại là K - không gian véc tơ chiều 2, điều này là vô lý.
1.2.9 Chú ý. Năm 2005, E. Diver, P. A. Leonard và K. S. Williams trong
bài báo Irreducible quartic polynomials with factorizations modulo p, Amer.
Math. Monthly, 112, No.10, 876-890, đã đưa ra điều kiện cần và đủ cho
đa thức bậc 4 với hệ số nguyên là bất khả quy trên Q nhưng khả quy
trên Zp với mọi số nguyên tố p. Kết quả này đã được Nguyễn Văn Lập
trình bày lại trong luận văn thạc sĩ của mình (xem [2]). Cũng năm 2005,
R. Guralnick, M. Schacher, J. Sonn trong bài báo Irreducible polynomials
which are locally reducible everywhere đã chỉ ra rằng, với mọi hợp số n ≥ 4,
tồn tại một đa thức bất khả quy f (x) ∈ Z[x] có bậc n mà khả quy trên
trường Zp với mọi số nguyên tố p.

15


Chương 2

Giá trị khả nghịch, giá trị nguyên
tố và tính bất khả quy
Mục tiêu thứ nhất của chương này là trình bày hai tiêu chuẩn bất khả
quy trên trường các số hữu tỷ Q liên quan đến các giá trị khả nghịch và
giá trị nguyên tố của đa thức với hệ số nguyên.
Mục tiêu thứ hai của chương này là trình bày một tiêu chuẩn mới về
tính bất khả quy trên trường Q của đa thức với hệ số nguyên sao cho các
hệ số tăng dần theo bậc và có hệ số cao nhất nguyên tố hoặc nhận ít nhất
một giá trị nguyên tố.
Các kết quả ở chương này một phần dựa theo bài báo [11] của R.

Thangadurai năm 2007 và một phần được viết dựa theo bài báo [8] của
A. Jakhar và N. Sangwan năm 2018: An irreducibility criterion for integer
polynomials, Amer. Math. Monthly, 125, 464-465.
Kết quả chính của Chương 2 là Định lý 6, Định lý 7, Định lý 8, Định
lý 9, Định lý 10, Định lý 11 và Định lý 12.

2.1

Giá trị khả nghịch và tính bất khả quy

Cho f (x) = an xn + . . . + a1 x + a0 có bậc n với hệ số nguyên. Ký hiệu
số lần đa thức f (x) nhận giá trị khả nghịch trên tập số nguyên là u(f ),
tức là
u(f ) := Card{m ∈ Z | f (m) ∈ {1, −1}}.
Chẳng hạn, nếu f (x) = x4 + 1, thì u(f ) = 1. Thật vậy, f (x) > 1 với mọi
x khác 0 và f (x) = 1 khi và chỉ khi x = 0. Nếu g(x) = x2 + x + 1 thì
16


u(g) = 2. Thật vậy, ta luôn có g(x) > 0 với mọi x. Hơn nữa, g(x) = 1 khi
và chỉ khi x = 0 hoặc x = −1.
Nếu f (x) nhận giá trị 1 tại các số nguyên x = bi với i = 1, 2, . . . , m,
m

(x − bi ), trong đó r(x) là đa thức với hệ số nguyên,

thì f (x) − 1 = r(x)

i=1


tức là

m

(x − bi ) + 1,

f (x) = r(x)
i=1

trong đó r(x) ∈ Z[x].
Tương tự, nếu f (x) nhận giá trị −1 tại các số nguyên x = bi với i =
m

(x − bi ), trong đó r(x) là đa thức với

1, 2, . . . , m, thì f (x) + 1 = r(x)

i=1

hệ số nguyên, tức là
m

(x − bi ) − 1,

f (x) = r(x)
i=1

trong đó r(x) ∈ Z[x].
2.1.1 Mệnh đề 4. Nếu f (x) nhận giá trị +1 (tương ứng −1) tại m > 3
giá trị nguyên khác nhau của biến x, thì f (x) không thể nhận giá trị −1

(tương ứng +1).
Chứng minh. Giả sử m > 3 và b1 , b2 , . . . , bm là các số nguyên đôi một phân
biệt sao cho f (bi ) = 1 với mọi i = 1, . . . , m. Khi đó

f (x) = (x − b1 )(x − b2 ) . . . (x − bm )g(x) + 1
với g(x) ∈ Z[x]. Giả sử bm+1 là số nguyên sao cho f (bm+1 ) = −1. Khi đó,
thay x = bm+1 vào đẳng thức trên ta nhận được

−1 = (bm+1 − b1 )(bm+1 − b2 ) . . . (bm+1 − bm )g(bm+1 ) + 1.
Suy ra

(bm+1 − b1 )(bm+1 − b2 ) . . . (bm+1 − bm )g(bm+1 ) = −2.
Do đó, các hiệu số bm+1 − bi là ước của −2, vì thế nó chỉ có thể là ±1 hoặc
±2. Vì các bi là đôi một phân biệt nên m ≤ 4. Nếu m = 4, thì ta có

(−1)(−2)(1)(2)g(bm+1 ) = −2.
17


1
Suy ra g(bm+1 ) = − , điều này là vô lý. Do đó m ≤ 3. Trường hợp còn lại
2
được chứng minh tương tự.
Chúng ta có thể xem chi tiết hơn Mệnh đề 4 trong một bài báo đăng
trên tạp chí nổi tiếng “Annals of Math.” xuất bản năm 1993 của hai nhà
toán học H. L. Dorwart và O. Ore.
2.1.2 Mệnh đề 5. Nếu f (x) có bậc n và n ≥ 4, thì u(f ) ≤ n.
Chứng minh. Giả sử u(f ) > n, n ≥ 4. Khi đó u(f ) ≥ 5. Suy ra f (x) nhận
giá trị 1 ít nhất 3 lần hoặc f (x) nhận giá trị −1 ít nhất 3 lần. Không mất
tính tổng quát ta có thể giả thiết f (x) nhận giá trị bằng 1 ít nhất 3 lần.

Giả sử f (x) nhận giá trị 1 lớn hơn 3 lần. Theo Mệnh đề 4, f (x) không
nhận giá trị −1, suy ra f (x) nhận giá trị 1 lớn hơn n lần, điều này là vô
lý vì deg f = n (đa thức có bậc n có nhiều nhất n nghiệm, suy ra đa thức
bậc n nhận cùng một giá trị tại nhiều nhất n điểm). Do đó f (x) nhận giá
trị bằng 1 tại đúng 3 lần. Giả sử f (x) nhận giá trị −1 tại m lần, thì

m = u(f ) − 3 > n − 3 ≥ 2.
Gọi b1 , b2 , b3 là các giá trị nguyên đôi một khác nhau sao cho

f (b1 ) = f (b2 ) = f (b3 ) = 1.
Gọi c1 , c2 , . . . cm , m ≥ 2 sao cho

f (c1 ) = f (c2 ) = · · · = f (cm ) = −1.
Suy ra

f (x) = (x − b1 )(x − b2 )(x − b3 )g(x) + 1,
với g(x) là đa thức có hệ số nguyên. Thay x = ci với i = 1, 2 ta có

−2 = (ci − b1 )(ci − b2 )(ci − b3 )g(ci ).
Không mất tính tổng quát ta giả thiết b1 < b2 < b3 . Khi đó ci − b1 , ci − b2 ,
ci − b3 là ba ước khác nhau của −2, vì thế một trong 3 ước đó phải là 2
hoặc −2, và hai ước còn lại là 1 và −1. Giả sử ứng với c1 , một trong ba
ước đó là 2. Khi đó c1 − b1 = 2, c1 − b2 = 1 và c1 − b3 = −1. Nếu ứng
với c2 , một trong các ước đó cũng là 2 thì ta phải có c2 − b1 = 2 và do
đó c2 = c1 , vô lý. Suy ra ứng với c2 , một trong các ước đó là −2. Suy ra
18


c2 − b3 = −2, do đó c2 = b2 , vô lý vì f (c2 ) = −1 trong khi đó f (b2 ) = 1.
Trường hợp ứng với c1 , một trong ba ước bằng −2, ta lập luận tương tự

và dẫn đến mâu thuẫn. Vậy mệnh đề được chứng minh.
2.1.3 Mệnh đề 6. Nếu f (x) là đa thức với hệ số nguyên có bậc n thì ta
luôn có u(f ) ≤ 2n.
Chứng minh. Gọi m1 là số lần f (x) nhận giá trị 1 và m2 là số lần f (x)
nhận giá trị −1. Khi đó u(f ) = m1 +m2 . Ta có mi ≤ n vì đa thức f (x)−1
và đa thức f (x) + 1 đều có không quá n nghiệm. Vì thế u(f ) ≤ 2n.
Định lý sau đậy là kết quả chính của mục này, đưa ra một tiêu chuẩn bất
khả quy trên Q của đa thức với hệ số nguyên dựa theo mối quan hệ giữa bậc
của đa thức và số lần nhận giá trị khả nghịch của đa thức. R. Thangadurai
trong bài báo Irreducibility of Polynomials Whose Coefficients are Integers,
đăng trên tạp chí Mathematics Newsletter tháng 9 năm 2007, ở trang 30,
đã phát biểu như sau.
2.1.4 Định lý 6. Cho f (x) là đa thức có hệ số nguyên. Nếu f (x) có bậc
n ≥ 8 và f (x) nhận giá trị 1 lớn hơn n/2 lần hoặc f (x) nhận giá trị −1
lớn hơn n/2 lần thì f (x) bất khả quy trên Q.
Chứng minh. Ta chứng minh phản chứng. Giả sử f (x) không bất khả quy
trên Q. Theo Bổ đề Gauss, f (x) = g(x)h(x) trong đó g(x) và h(x) là các
nhân tử không tầm thường của f (x), tức là g(x) và h(x) đều là đa thức
bậc dương với hệ số nguyên. Vì bậc của f (x) là tổng của bậc của g(x) và
h(x), nên không mất tính tổng quát ta có thể giả thiết g(x) có bậc m với
m ≥ n/2 ≥ 4. Theo giả thiết, f (x) nhận giá trị 1 tại u(f ) lần hoặc f (x)
nhận giá trị −1 tại u(f ) lần. Không mất tính tổng quát ta có thể giả thiết
f (x) nhận giá trị −1 tại u(f ) lần.
Ta khẳng định g(x) nhận giá trị 1 tại u(g) lần hoặc g(x) nhận giá trị
−1 tại u(g) lần. Nếu g(x) nhận giá trị 1 tại ít nhất 4 lần hoặc g(x) nhận
giá trị −1 tại ít nhất 4 lần thì khẳng định trên suy ra ngay từ Mệnh đề 4.
Giả sử ngược lại, tức là g(x) nhận giá trị 1 nhỏ hơn 4 lần và cũng nhận
giá trị −1 nhỏ hơn 4 lần. Chú ý rằng u(g) ≥ u(f ) và u(h) ≥ u(f ) bởi vì
nếu f (x) nhận giá trị ±1 tại x = a thì g(x) và h(x) cũng nhận giá trị ±1
tại x = a. Do đó theo giả thiết ta có u(g) ≥ u(f ) > n/2. Suy ra u(g) ≥ 5.

Không mất tính tổng quát ta có thể giả thiết g(x) nhận giá trị 1 tại đúng
19


3 lần. Khi đó g(x) nhận giá trị −1 tại ít nhất 2 lần. Theo lập luận tương
tự như trong chứng minh Mệnh đề 5, ta thấy điều này không thể xảy ra.
Vậy khẳng định được chứng minh.
Theo khẳng định trên, ta có thể giả thiết g(x) nhận giá trị 1 tại u(g)
lần với u(g) > n/2. Gọi a1 , a2 , . . . au(f ) là các số nguyên phân biệt sao
cho f (ai ) = 1 với mọi i. Theo khẳng định trên g(ai ) = 1 với mọi i. Vì
f (ai ) = g(ai )h(ai ) nên h(ai ) = 1 với mọi i. Như vậy h(x) nhận giá trị
bằng 1 tại ít nhất u(f ) lần. Vì u(f ) > n/2 nên deg h(x) ≥ u(f ) > n/2.
Suy ra deg h(x) + deg g(x) > deg f (x). Điều này là vô lý.
Chứng minh tương tự, nếu g(x) nhận giá trị −1 lớn hơn n/2 lần, thì
h(x) cũng nhận giá trị −1 lớn hơn n/2 lần, và vì thế tổng bậc của g(x) và
bậc của h(x) lớn hơn n, vô lý. Do vậy f (x) là bất khả quy trên Q.
2.1.5 Ví dụ 6. Xét tính bất khả quy của các đa thức sau trên Q:
(i) f (x) = x9 − 13x7 + 37x5 − 13x3 + 36x + 1.
(ii) g(x) = (x − 1)(x2 − 1) − 1.
Chứng minh.
(i) Ta có f (x) − 1 = (x − 2)(x + 2)(x − 3)(x + 3)x(x4 + 1). Vì thế f (x)
nhận giá trị 1 tại đúng 5 điểm. Vì deg f (x) = 9 và 5 > 9/2 nên theo Định
lý 6 ta suy ra f (x) bất khả quy trên Q.
(ii) Ta có g(x) + 1 = (x − 1)(x2 − 1). Vì thế g(x) nhận giá trị −1 tại đúng
2 điểm. deg g(x) = 3 và 2 > 3/2, nhưng g(x) không bất khả quy trên Q
theo Định lý 6. Thật vậy, ta có g(0) = 0 do đó g(x) khả quy trên Q.
Rõ ràng đa thức g(x) trong Ví dụ 6 có số lần nhận giá trị −1 lơn hơn
deg g(x)/2 nhưng không bất khả quy theo Định lý 6 bởi vì deg g(x) < 8.
Chú ý rằng việc xét tính bất khả quy của đa thức f (x) nói trên bằng
Tiêu chuẩn Eisenstein hoặc bằng phương pháp rút gọn theo module một

số nguyên tố đều không khả thi. Tuy nhiên nếu sử dụng Định lý 6 thì
việc xét tính bất khả quy của đa thức này quy về việc phân tích đa thức
f (x) − 1 thành nhân tử, và đa thức f (x) − 1 có 5 nghiệm nguyên nên việc
phân tích là dễ dàng.
Trong phần cuối của mục này, chúng tôi trình bày khái niệm và một số
kết quả về đa thức béo liên quan đến giá trị khả nghịch của đa thức.
20


2.1.6 Định nghĩa 2. Đa thức g(x) ∈ Z[x] được gọi là béo nếu

(g) := u(g) − deg g(x) > 0.
Chẳng hạn, đa thức f (x) = 2x2 −4x+1 là béo vì deg f (x) = 2 và u(f ) = 3.
Đa thức g(x) = x2 + 3x + 1 là đa thức béo vì deg g(x) = 2 và u(g) = 4.
Đa thức h(x) = x4 + 3x2 + 2 không béo vì deg h(x) = 4 và u(h) = 0.
Đa thức t(x) = x3 − x2 + x + 1 không béo vì deg t(x) = 3 và u(t) = 1.
2.1.7 Mệnh đề 7. Nếu f (x) là đa thức béo, thì deg f (x) ≤ 3.
Chứng minh. Giả sử deg f (x) ≥ 4.
Theo Mệnh đề 5, ta có u(f ) ≤ deg f (x). Mà f (x) là đa thức béo nên
u(f ) > deg f (x). Do đó deg f (x) ≤ 3.
Năm 1993, hai nhà toán học H.L. Dorwart và O. Ore trong bài báo
Criteria for the irreducibility of polynomial, Annals of Math. 34, No. 1,
81-94, đã chứng minh rằng nếu f (x) là một đa thức béo có bậc n, thì
f (x) = ±hi (±x ± a), trong đó a là một số nguyên và hi (x), i = 1, 2, . . . , 5
được cho dưới đây

h1 (x) = x(x − 1)(x − 3) + 1, n = 3, u(f ) = 4.
h2 (x) = (x − 1)(x − 2) − 1, n = 2, u(f ) = 4.
h3 (x) = 2x(x − 2) + 1, n = 2, u(f ) = 3.
h4 (x) = 2x − 1, n = 1, u(f ) = 2.

h5 (x) = x − 1, n = 1, u(f ) = 2.
2.2

Giá trị nguyên tố và tính bất khả quy

Mục tiêu của phần này là trình bày một số tiêu chuẩn bất khả quy trên
Q của đa thức với hệ số nguyên trong mối quan hệ với giá trị nguyên tố
của đa thức.
Kí hiệu

P (f ) := Card{n ∈ Z : f (n) = ±p, trong đó p là số nguyên tố}.
Khi đó P (f ) là số lần f (x) nhận giá trị nguyên tố hoặc số đối của số
nguyên tố. Chú ý rằng P (f ) = ∞ với vô hạn đa thức f (x) ∈ Z[x]. Thật
vậy, Định lý Dirichlet phát biểu rằng nếu a, b là hai số tự nhiên nguyên tố
21


cùng nhau, thì tồn tại vô hạn số nguyên tố trong dãy số {an + b}n∈N . Vì
vậy có vô hạn đa thức dạng f (x) = ax + b với a, b là các số nguyên sao
cho gcd(a, b) = 1, và các đa thức này thỏa mãn P (f ) = ∞.
Định lý sau đây là một trong hai kết quả chính của mục này, chỉ ra
rằng nếu f (x) nhận giá trị nguyên tố tại nhiều hơn 2 lần bậc của nó
thì f (x) bất khả quy trên Q. Định lý này được viết dựa theo phát biểu
của R. Thangadurai trong bài báo Irreducibility of Polynomials Whose
Coefficients are Integers, Mathematics Newsletter, Vol 17, ở trang 31. Tiêu
chuẩn này theo nghĩa nào đó là khá dễ sử dụng để xét tính bất khả quy.
2.2.1 Định lý 7. Cho f (x) là đa thức có hệ số nguyên với bậc là n > 0.
Nếu P (f ) > 2n thì f (x) bất khả quy trên Q.
Chứng minh. Ta chứng minh bằng phản chứng. Giả sử f (x) không bất
khả quy. Theo Bổ đề Gauss, ta có phân tích f (x) = g(x)h(x) trong đó

g(x), h(x) ∈ Z[x] và bậc của g(x), bậc của h(x) đều dương. Giả sử P (f ) =
m. Khi đó tồn tại các số nguyên b1 , b2 , . . . , bm sao cho f (bi ) là số nguyên
tố hoặc là số đối của số nguyên tố.
Giả sử p = f (n) = g(n)h(n) là một số nguyên tố. Khi đó, hoặc g(n) =
±1 hoặc h(n) = ±1. Do vậy, g(bi ) = ±1 hoặc h(bi ) = ±1 với mọi i =
1, 2, . . . , m. Suy ra u(h) + u(g) ≥ P (f ) > 2n theo giả thiết. Giả sử
deg g(x) = r. Khi đó deg h(x) = n−r. Theo Mệnh đề 2.1.3, ta có u(g) ≤ 2r
và u(h) ≤ 2(n − r). Suy ra u(g) + u(h) ≤ 2n, điều này là vô lý. Do đó
f (x) bất khả quy.
Hệ quả sau đây được suy ra trực tiếp từ Định lý 7.
2.2.2 Hệ quả 1. Cho f (x) là đa thức có hệ số nguyên với bậc là n > 0.
Nếu P (f ) = ∞ thì f (x) bất khả quy trên Q.
2.2.3 Ví dụ 7. Các đa thức sau là bất khả quy trên Q.
(i) f (x) = x4 − 10x2 + 11.
(ii) g(x) = x4 − 3x − 3.
(iii) h(x) = 3x2 + 11x + 121.
Chứng minh.
(i) Ta có f (0) = 2, f (1) = 2, f (−1) = 2, f (3) = 2, f (−3) = 2,
f (2) = 17, f (−2) = 17, f (4) = 107, f (−4) = 107. Vì 2, 17, 107 là các số
22


nguyên tố nên f (x) nhận giá trị nguyên tố tại ít nhất 9 lần. Vì bậc của đa
thức là 4 nên theo Định lý 7, đa thức f (x) bất khả quy trên Q.
(ii) Ta có g(0) = 3, g(1) = −5, g(2) = 7, g(−2) = 19, g(4) = 241,
g(−4) = 271, g(5) = 613, g(−5) = 643, g(7) = 2377.
Vì 3, −5, 7, 19, 241, 271, 613, 643, 2377 là các số nguyên tố hoặc số đối
của số nguyên tố và deg g(x) = 4 nên theo Định lý 7, đa thức g(x) là bất
khả quy trên Q.
(iii) Ta có h(−7) = 191, h(−6) = 163, h(−1) = 113, h(3) = 181, h(5) =

251. Vì 113, 163, 181, 191, 251 là các số nguyên tố nên h(x) nhận giá trị
nguyên tố tại ít nhất 5 lần. Vì bậc của đa thức là 2 nên theo Định lý 7,
đa thức h(x) bất khả quy trên Q.
Chú ý rằng, đa thức g(x) trong Ví dụ 7 là bất khả quy theo Tiêu chuẩn
Eisenstein với p = 3. Tính bất khả quy của h(x) có thể suy ra từ thực tế
h(x) không có nghiệm hữu tỷ (hệ số tự do của h(x) có các ước: ±1, ±11,
±121 nên việc tìm nghiệm của h(x) tương đương với việc tìm giá trị của
h(x) tại những ước này). Tuy nhiên tính bất khả quy của đa thức f (x)
trong Ví dụ 7 không thể suy ra từ Tiêu chuẩn Eisenstein và cũng không
thể suy ra từ phương pháp rút gọn theo module một số nguyên tố.
Định lý sau đây cũng là kết quả được viết dựa theo nội dung bài báo
của R. Thangadurai Irreducibility of Polynomials Whose Coefficients are
Integers, Mathematics Newsletter, Vol 17, ở trang 31. Đây là kết quả chính
thứ hai của mục này, cho ta một tiêu chuẩn bất khả quy của đa thức dựa
trên số lần nhận giá trị nguyên tố, giá trị đối nguyên tố hoặc giá trị khả
nghịch. Theo nhiều nghĩa nào đó, đây cũng là một tiêu chuẩn hữu hiệu để
xét tính bất khả quy của đa thức.
2.2.4 Định lý 8. Cho f (x) là đa thức có bậc n với hệ số nguyên. Nếu tồn
tại n + 1 số nguyên m1 , m2 , . . . , mn+1 sao cho |mi − mj | > 2 với mọi i = j
và f (mi ) là số nguyên tố, số đối của số nguyên tố hoặc số khả nghịch, thì
f (x) bất khả quy trên Q.
Chứng minh. Ta chứng minh bằng phản chứng. Giả sử f (x) không bất
khả quy trên Q. Theo Bổ đề Gauss, f (x) = g(x)h(x), trong đó g(x), h(x)
là các đa thức bậc dương có hệ số nguyên. Xét tại các số nguyên mi , vì
f (mi ) = g(mi )h(mi ) là số nguyên tố, số đối của số nguyên tố hoặc khả
23


×